Quantcast
  • Register
PhysicsOverflow is a next-generation academic platform for physicists and astronomers, including a community peer review system and a postgraduate-level discussion forum analogous to MathOverflow.

Welcome to PhysicsOverflow! PhysicsOverflow is an open platform for community peer review and graduate-level Physics discussion.

Please help promote PhysicsOverflow ads elsewhere if you like it.

News

PO is now at the Physics Department of Bielefeld University!

New printer friendly PO pages!

Migration to Bielefeld University was successful!

Please vote for this year's PhysicsOverflow ads!

Please do help out in categorising submissions. Submit a paper to PhysicsOverflow!

... see more

Tools for paper authors

Submit paper
Claim Paper Authorship

Tools for SE users

Search User
Reclaim SE Account
Request Account Merger
Nativise imported posts
Claim post (deleted users)
Import SE post

Users whose questions have been imported from Physics Stack Exchange, Theoretical Physics Stack Exchange, or any other Stack Exchange site are kindly requested to reclaim their account and not to register as a new user.

Public \(\beta\) tools

Report a bug with a feature
Request a new functionality
404 page design
Send feedback

Attributions

(propose a free ad)

Site Statistics

205 submissions , 163 unreviewed
5,047 questions , 2,200 unanswered
5,345 answers , 22,709 comments
1,470 users with positive rep
816 active unimported users
More ...

  What happens to the Lagrangian of the Dirac theory under charge conjugation?

+ 3 like - 0 dislike
5776 views

Consider a charge conjugation operator which acts on the Dirac field($\psi$) as
$$\psi_{C} \equiv \mathcal{C}\psi\mathcal{C}^{-1} = C\gamma_{0}^{T}\psi^{*}$$ Just as we can operate the parity operator on the Lagrangian, and we say that a theory has a symmetry if $$\mathcal{P}\mathcal{L}(t,x^{i})\mathcal{P}^{-1} = \mathcal{L}(t,-x^{i})$$

Suppose we operate $\mathcal{C}$ on the Dirac Lagrangian what should we get? $$\mathcal{C}\mathcal{L}_{Dirac}(x^{\mu})\mathcal{C}^{-1} = \mathcal{L}^{*}_{Dirac}(x^{\mu}) ?$$ in analogy to the transformation of the scalar field $\phi$ under charge conjugation.

On a same note one can ask what equation should $\psi_{C}$ satisfy? Should it satisfy the conjugated Dirac equation as $$(i\gamma^{\mu}\partial_{\mu} + m)\psi_{C} = 0 ?$$ If so can someone give me the physical interpretation for it.

I am asking this question as I want to explicitly use $\psi_{C}$ and check whether it keeps the Dirac Lagrangian invariant. I have done a calculation by substituting $\psi_{C}$ in the Dirac equation and have found it is not satisfying as shown below.

$$(i\gamma_{\mu}\partial_{\mu} - m)\psi_{C} = i(\gamma_{\mu}C\gamma_{0}^{T})\partial^{\mu}\psi^{*} - (C\gamma_{0}^{T})m\psi^{*}$$ We will use $C^{-1}\gamma_{\mu}C = - \gamma_{\mu}^{T}$ and $\{\gamma_{\mu}^{T},\gamma_{\nu}^{T}\} = 2g_{\mu\nu}$.
Consider \begin{align} \gamma_{\mu}C\gamma_{0}^{T} &= CC^{-1}\gamma_{\mu}C\gamma_{0}^{T} \\ &= -C\gamma_{\mu}^{T}\gamma_{0}^{T} \\ &= C\gamma_{0}^{T}\gamma_{\mu}^{T} \end{align} Hence substituting back we will get
\begin{align} (i\gamma_{\mu}\partial_{\mu} - m)\psi_{C} &= C\gamma_{0}^{T}(i\gamma_{\mu}^{T}\partial^{\mu}\psi^* - m\psi^*) \\ &= C\gamma_{0}^{T}[(i\gamma_{\mu}^{T}\partial^{\mu}\psi^* - m\psi^*)^{T}]^{T} \\ &= C\gamma_{0}^{T}(i\psi^{\dagger}\gamma_{\mu}\partial^{\mu} - m\psi^{\dagger})^{T} \\ &= C\gamma_{0}^{T}[(i\bar{\psi}\gamma_{0}\gamma_{\mu}\partial^{\mu} - m\bar{\psi}\gamma_{0})]^{T} \\ \end{align} Now \begin{align} \gamma_{0}\gamma_{\mu}\partial^{\mu} &= (\gamma_{0}\partial^{t} + \gamma_{i}\partial^{i})\gamma_{0} \end{align} If we substitute back we will get \begin{align} (i\gamma_{\mu}\partial_{\mu} - m)\psi_{C} &= C\gamma_{0}^{T}[\{i\bar{\psi}(\gamma_{0}\partial^{t} + \gamma_{i}\partial^{i}) - m\bar{\psi}\}\gamma_{0}]^T \\ &\neq 0 \end{align}

This post imported from StackExchange Physics at 2014-07-13 04:41 (UCT), posted by SE-user Omkar
asked Oct 22, 2012 in Theoretical Physics by Omkar (15 points) [ no revision ]
Good question :-)

This post imported from StackExchange Physics at 2014-07-13 04:41 (UCT), posted by SE-user David Z

1 Answer

+ 3 like - 0 dislike

The short answer to the question, "What happens to the Lagrangian of the Dirac theory under charge conjugation?" is, "Nothing." It is invariant with respect to charge conjugation.

Before getting to the longer exposition, I'd like to point out a potential misunderstanding about the nature of invariance of the equations of motion under symmetry transformations that arises about your statement regarding the parity ($\mathbf{x}\to-\mathbf{x}$). Your equation \begin{align} \mathcal{P}\mathcal{L}(t,x^{i})\mathcal{P}^{-1} = \mathcal{L}(t,-x^{i}) \end{align} is correct. But this, in and of itself, doesn't "say that a theory has a symmetry." In fact, it places a restriction on the theory -- that the theory must be an even function of the position vector. For example, for a real scalar field, $\phi(t,\mathbf{x})$, the kinetic energy operator in the Lagrangian density, $\partial_\mu \partial^\mu \phi(x)$ is invariant under parity. In fact, it is only the action ($S=\int d^4x \mathcal{L}$) that need be invariant under the symmetry transformation. (Often this reduces to the invariance of the Lagrangian density.) So each term of the Lagrangian (density) need not be invariant (though this is often the case).

Returning to the Dirac equation -- the full statement, in words, of invariance of the theory of the interaction of electrons with light (QED) under the discrete transformations ($\mathcal{P},\mathcal{C}$, & $\mathcal{T}$) is that the theory is invariant under each of them separately or in any combination. (QED is less "interesting" than the electroweak theory in this regard since the electroweak theory appears to violate all three of these separately -- but perhaps not all simultaneously.)

We have to remember that the invariance under $\mathcal{C}$ requires the transformation not only of the wave function, $\psi_{C} \equiv \mathcal{C}\psi\mathcal{C}^{-1} = C\gamma_{0}^{T}\psi^{*}$ but also the charge, $q\to -q$. In the case of the free Dirac equation/Lagrangian considered above, the charge does not feature, so it's not directly relevant to the present discussion but it's important to keep in mind.

Now the direct answers to your questions. (I won't do the algebra since, if you can carry out the calculations for the Dirac equation itself, then the transformation of the Lagrangian should be straightforward.)

"Suppose we operate C on the Dirac Lagrangian what should we get?" The corrected relation is:

\begin{align}\mathcal{C}\mathcal{L}_{Dirac}(x^{\mu})\mathcal{C}^{-1} = \mathcal{L}_{Dirac}(x^{\mu})\end{align}

(Incidentally, your relation turns out to be all right since the Lagrangian (density) must be a Hermitian, scalar operator, so $\mathcal{L}^* = \mathcal{L}^\dagger = \mathcal{L}$. EDIT: Thanks to Omkar for pointing out that this is wrong. $\mathcal{L}^*\ne\mathcal{L}$.)

On a same note one can ask what equation should C satisfy? Should it satisfy the conjugated Dirac equation as \begin{align} > (i\gamma^{\mu}\partial_{\mu} + m)\psi_{C} = 0? \end{align}

If you're using the "West Coast" metric $(+1,-1,-1,-1)$ then the equation that $\psi_C$ should satisfy is the one above with $m\to -m$. That is, the free Dirac equation is the same for $\psi$ and $\psi_C$. This is because the masses of the particle and anti-particle are identical, as first divined by Dirac. (If you're using the metric of the opposite sign, then you're equation is correct.)

This post imported from StackExchange Physics at 2014-07-13 04:41 (UCT), posted by SE-user MarkWayne
answered Oct 22, 2012 by MarkWayne (270 points) [ no revision ]
Thank You for your reply. I agree with your argument regarding the Lagrangian. And even your argument regarding the form of the equation sounds correct and something we should expect given that the Lagrangian is same. However I did a calculation and I found otherwise. I will put up the calculation in the question, maybe I am making some error.

This post imported from StackExchange Physics at 2014-07-13 04:41 (UCT), posted by SE-user Omkar
another point is that even though $\mathcal{L}$ is hermitian, I think it is not real, as the term $\bar{\psi}\psi$ is not real. Hence you can't say $\mathcal{L}^* = \mathcal{L}$

This post imported from StackExchange Physics at 2014-07-13 04:41 (UCT), posted by SE-user Omkar
I see an error in the above. You should have: $\gamma_{0} \gamma_{\mu}\partial^{\mu} = (\gamma_{0}\partial^{t} - \gamma_{i}\partial^{i})\gamma_{0}$. I wouldn't do it this way though. I get the correct result by starting with the Dirac equation for $\psi$, taking the complex conjugate and showing that $C \propto \gamma^2$ (in what's sometimes called the Dirac representation of the $\gamma$ matrices). And you're right that $\mathcal{L}^*\ne\mathcal{L}$. Sorry for that.

This post imported from StackExchange Physics at 2014-07-13 04:41 (UCT), posted by SE-user MarkWayne
Sorry again -- $C$ should be $\propto \gamma^2\gamma^0$. (I'm using Bjorken & Drell conventions.)

This post imported from StackExchange Physics at 2014-07-13 04:41 (UCT), posted by SE-user MarkWayne
Ya you are right my notations are bad, I am using a very bad book :\ . Btw $\gamma_{0}\gamma_{\mu}\partial^{\mu} = \gamma_{0}\gamma_{0}\partial^{t} - \gamma_{0}\gamma_{i}\partial^{i} = \gamma_{0}\gamma_{0}\partial^{t} + \gamma_{i}\gamma_{0}\partial^{i} = (\gamma_{0}\partial^{t} + \gamma_{i}\partial^{i})\gamma_{0}$

This post imported from StackExchange Physics at 2014-07-13 04:41 (UCT), posted by SE-user Omkar
Omkar, this is a common mistake you've made in the above line for $\gamma_\mu\partial^\mu$. Consider $a_\mu b^\mu = a_0 b^0 + a_i b^i$ -- there's no minus sign since you haven't lowered the index yet. The correct relation is: $\gamma_\mu\partial^\mu = \gamma_0 \partial_0 - \sum_{i=1}^3 \gamma_i \partial_i$.

This post imported from StackExchange Physics at 2014-07-13 04:41 (UCT), posted by SE-user MarkWayne

Why $\mathcal{L}^*\neq \mathcal{L}$? Since Lagrangian is also a scalar, I think it's free if one wants to do a transpose which makes Hermitian equal to complex conjugate.

Your answer

Please use answers only to (at least partly) answer questions. To comment, discuss, or ask for clarification, leave a comment instead.
To mask links under text, please type your text, highlight it, and click the "link" button. You can then enter your link URL.
Please consult the FAQ for as to how to format your post.
This is the answer box; if you want to write a comment instead, please use the 'add comment' button.
Live preview (may slow down editor)   Preview
Your name to display (optional):
Privacy: Your email address will only be used for sending these notifications.
Anti-spam verification:
If you are a human please identify the position of the character covered by the symbol $\varnothing$ in the following word:
$\varnothing\hbar$ysicsOverflow
Then drag the red bullet below over the corresponding character of our banner. When you drop it there, the bullet changes to green (on slow internet connections after a few seconds).
Please complete the anti-spam verification




user contributions licensed under cc by-sa 3.0 with attribution required

Your rights
...